Computation of a double exponential integral












1












$begingroup$


I want to understand the behavior of this integral
$$ int_0^x e^{-frac{c}{t^2}} frac{1}{t^5} e^{-frac{c_2}{(x-t)^2}} frac{1}{(x-t)^5} dt. $$
The ideal answer would be a way to explicitly compute this, but I have no clue. This integral is clearly definite, but my problem is to understand quantitatively its behavior for small and large values of $c$ and $c_2$. Any idea on how to proceed?










share|cite|improve this question











$endgroup$












  • $begingroup$
    At first i would simplify the integrand!
    $endgroup$
    – Dr. Sonnhard Graubner
    Jan 19 at 12:32










  • $begingroup$
    What would be the best way to do so? The two obvious changes of variable ($t to frac{1}{t}$ or $t to frac{1}{x-t}$) seem to lead to something more complicated.
    $endgroup$
    – Gâteau-Gallois
    Jan 19 at 12:44












  • $begingroup$
    Why did you delete the second part of the question ?
    $endgroup$
    – Claude Leibovici
    Jan 19 at 13:14










  • $begingroup$
    I think the integral I had was not convergent, near 0 it is homogeneous to $1/t^k$ for $k geq 2$... I need to check again the entire series computation
    $endgroup$
    – Gâteau-Gallois
    Jan 19 at 13:38
















1












$begingroup$


I want to understand the behavior of this integral
$$ int_0^x e^{-frac{c}{t^2}} frac{1}{t^5} e^{-frac{c_2}{(x-t)^2}} frac{1}{(x-t)^5} dt. $$
The ideal answer would be a way to explicitly compute this, but I have no clue. This integral is clearly definite, but my problem is to understand quantitatively its behavior for small and large values of $c$ and $c_2$. Any idea on how to proceed?










share|cite|improve this question











$endgroup$












  • $begingroup$
    At first i would simplify the integrand!
    $endgroup$
    – Dr. Sonnhard Graubner
    Jan 19 at 12:32










  • $begingroup$
    What would be the best way to do so? The two obvious changes of variable ($t to frac{1}{t}$ or $t to frac{1}{x-t}$) seem to lead to something more complicated.
    $endgroup$
    – Gâteau-Gallois
    Jan 19 at 12:44












  • $begingroup$
    Why did you delete the second part of the question ?
    $endgroup$
    – Claude Leibovici
    Jan 19 at 13:14










  • $begingroup$
    I think the integral I had was not convergent, near 0 it is homogeneous to $1/t^k$ for $k geq 2$... I need to check again the entire series computation
    $endgroup$
    – Gâteau-Gallois
    Jan 19 at 13:38














1












1








1





$begingroup$


I want to understand the behavior of this integral
$$ int_0^x e^{-frac{c}{t^2}} frac{1}{t^5} e^{-frac{c_2}{(x-t)^2}} frac{1}{(x-t)^5} dt. $$
The ideal answer would be a way to explicitly compute this, but I have no clue. This integral is clearly definite, but my problem is to understand quantitatively its behavior for small and large values of $c$ and $c_2$. Any idea on how to proceed?










share|cite|improve this question











$endgroup$




I want to understand the behavior of this integral
$$ int_0^x e^{-frac{c}{t^2}} frac{1}{t^5} e^{-frac{c_2}{(x-t)^2}} frac{1}{(x-t)^5} dt. $$
The ideal answer would be a way to explicitly compute this, but I have no clue. This integral is clearly definite, but my problem is to understand quantitatively its behavior for small and large values of $c$ and $c_2$. Any idea on how to proceed?







analysis definite-integrals






share|cite|improve this question















share|cite|improve this question













share|cite|improve this question




share|cite|improve this question








edited Jan 19 at 13:00







Gâteau-Gallois

















asked Jan 19 at 12:30









Gâteau-GalloisGâteau-Gallois

362113




362113












  • $begingroup$
    At first i would simplify the integrand!
    $endgroup$
    – Dr. Sonnhard Graubner
    Jan 19 at 12:32










  • $begingroup$
    What would be the best way to do so? The two obvious changes of variable ($t to frac{1}{t}$ or $t to frac{1}{x-t}$) seem to lead to something more complicated.
    $endgroup$
    – Gâteau-Gallois
    Jan 19 at 12:44












  • $begingroup$
    Why did you delete the second part of the question ?
    $endgroup$
    – Claude Leibovici
    Jan 19 at 13:14










  • $begingroup$
    I think the integral I had was not convergent, near 0 it is homogeneous to $1/t^k$ for $k geq 2$... I need to check again the entire series computation
    $endgroup$
    – Gâteau-Gallois
    Jan 19 at 13:38


















  • $begingroup$
    At first i would simplify the integrand!
    $endgroup$
    – Dr. Sonnhard Graubner
    Jan 19 at 12:32










  • $begingroup$
    What would be the best way to do so? The two obvious changes of variable ($t to frac{1}{t}$ or $t to frac{1}{x-t}$) seem to lead to something more complicated.
    $endgroup$
    – Gâteau-Gallois
    Jan 19 at 12:44












  • $begingroup$
    Why did you delete the second part of the question ?
    $endgroup$
    – Claude Leibovici
    Jan 19 at 13:14










  • $begingroup$
    I think the integral I had was not convergent, near 0 it is homogeneous to $1/t^k$ for $k geq 2$... I need to check again the entire series computation
    $endgroup$
    – Gâteau-Gallois
    Jan 19 at 13:38
















$begingroup$
At first i would simplify the integrand!
$endgroup$
– Dr. Sonnhard Graubner
Jan 19 at 12:32




$begingroup$
At first i would simplify the integrand!
$endgroup$
– Dr. Sonnhard Graubner
Jan 19 at 12:32












$begingroup$
What would be the best way to do so? The two obvious changes of variable ($t to frac{1}{t}$ or $t to frac{1}{x-t}$) seem to lead to something more complicated.
$endgroup$
– Gâteau-Gallois
Jan 19 at 12:44






$begingroup$
What would be the best way to do so? The two obvious changes of variable ($t to frac{1}{t}$ or $t to frac{1}{x-t}$) seem to lead to something more complicated.
$endgroup$
– Gâteau-Gallois
Jan 19 at 12:44














$begingroup$
Why did you delete the second part of the question ?
$endgroup$
– Claude Leibovici
Jan 19 at 13:14




$begingroup$
Why did you delete the second part of the question ?
$endgroup$
– Claude Leibovici
Jan 19 at 13:14












$begingroup$
I think the integral I had was not convergent, near 0 it is homogeneous to $1/t^k$ for $k geq 2$... I need to check again the entire series computation
$endgroup$
– Gâteau-Gallois
Jan 19 at 13:38




$begingroup$
I think the integral I had was not convergent, near 0 it is homogeneous to $1/t^k$ for $k geq 2$... I need to check again the entire series computation
$endgroup$
– Gâteau-Gallois
Jan 19 at 13:38










2 Answers
2






active

oldest

votes


















2












$begingroup$

Hint



Assuming $t<x$, $x>0$,$$intfrac{1}{t^k} frac{1}{(x-t)^l},dt=x^{-k-l+1} B_{frac{t}{x}}(1-k,1-l)$$






share|cite|improve this answer









$endgroup$





















    0












    $begingroup$

    I mean this here $$e^{-frac{c}{t^2}-frac{c_2}{(x-t)^2}}cdot frac{1}{(t(x-t))^5}$$






    share|cite|improve this answer









    $endgroup$













      Your Answer





      StackExchange.ifUsing("editor", function () {
      return StackExchange.using("mathjaxEditing", function () {
      StackExchange.MarkdownEditor.creationCallbacks.add(function (editor, postfix) {
      StackExchange.mathjaxEditing.prepareWmdForMathJax(editor, postfix, [["$", "$"], ["\\(","\\)"]]);
      });
      });
      }, "mathjax-editing");

      StackExchange.ready(function() {
      var channelOptions = {
      tags: "".split(" "),
      id: "69"
      };
      initTagRenderer("".split(" "), "".split(" "), channelOptions);

      StackExchange.using("externalEditor", function() {
      // Have to fire editor after snippets, if snippets enabled
      if (StackExchange.settings.snippets.snippetsEnabled) {
      StackExchange.using("snippets", function() {
      createEditor();
      });
      }
      else {
      createEditor();
      }
      });

      function createEditor() {
      StackExchange.prepareEditor({
      heartbeatType: 'answer',
      autoActivateHeartbeat: false,
      convertImagesToLinks: true,
      noModals: true,
      showLowRepImageUploadWarning: true,
      reputationToPostImages: 10,
      bindNavPrevention: true,
      postfix: "",
      imageUploader: {
      brandingHtml: "Powered by u003ca class="icon-imgur-white" href="https://imgur.com/"u003eu003c/au003e",
      contentPolicyHtml: "User contributions licensed under u003ca href="https://creativecommons.org/licenses/by-sa/3.0/"u003ecc by-sa 3.0 with attribution requiredu003c/au003e u003ca href="https://stackoverflow.com/legal/content-policy"u003e(content policy)u003c/au003e",
      allowUrls: true
      },
      noCode: true, onDemand: true,
      discardSelector: ".discard-answer"
      ,immediatelyShowMarkdownHelp:true
      });


      }
      });














      draft saved

      draft discarded


















      StackExchange.ready(
      function () {
      StackExchange.openid.initPostLogin('.new-post-login', 'https%3a%2f%2fmath.stackexchange.com%2fquestions%2f3079311%2fcomputation-of-a-double-exponential-integral%23new-answer', 'question_page');
      }
      );

      Post as a guest















      Required, but never shown

























      2 Answers
      2






      active

      oldest

      votes








      2 Answers
      2






      active

      oldest

      votes









      active

      oldest

      votes






      active

      oldest

      votes









      2












      $begingroup$

      Hint



      Assuming $t<x$, $x>0$,$$intfrac{1}{t^k} frac{1}{(x-t)^l},dt=x^{-k-l+1} B_{frac{t}{x}}(1-k,1-l)$$






      share|cite|improve this answer









      $endgroup$


















        2












        $begingroup$

        Hint



        Assuming $t<x$, $x>0$,$$intfrac{1}{t^k} frac{1}{(x-t)^l},dt=x^{-k-l+1} B_{frac{t}{x}}(1-k,1-l)$$






        share|cite|improve this answer









        $endgroup$
















          2












          2








          2





          $begingroup$

          Hint



          Assuming $t<x$, $x>0$,$$intfrac{1}{t^k} frac{1}{(x-t)^l},dt=x^{-k-l+1} B_{frac{t}{x}}(1-k,1-l)$$






          share|cite|improve this answer









          $endgroup$



          Hint



          Assuming $t<x$, $x>0$,$$intfrac{1}{t^k} frac{1}{(x-t)^l},dt=x^{-k-l+1} B_{frac{t}{x}}(1-k,1-l)$$







          share|cite|improve this answer












          share|cite|improve this answer



          share|cite|improve this answer










          answered Jan 19 at 13:09









          Claude LeiboviciClaude Leibovici

          123k1157134




          123k1157134























              0












              $begingroup$

              I mean this here $$e^{-frac{c}{t^2}-frac{c_2}{(x-t)^2}}cdot frac{1}{(t(x-t))^5}$$






              share|cite|improve this answer









              $endgroup$


















                0












                $begingroup$

                I mean this here $$e^{-frac{c}{t^2}-frac{c_2}{(x-t)^2}}cdot frac{1}{(t(x-t))^5}$$






                share|cite|improve this answer









                $endgroup$
















                  0












                  0








                  0





                  $begingroup$

                  I mean this here $$e^{-frac{c}{t^2}-frac{c_2}{(x-t)^2}}cdot frac{1}{(t(x-t))^5}$$






                  share|cite|improve this answer









                  $endgroup$



                  I mean this here $$e^{-frac{c}{t^2}-frac{c_2}{(x-t)^2}}cdot frac{1}{(t(x-t))^5}$$







                  share|cite|improve this answer












                  share|cite|improve this answer



                  share|cite|improve this answer










                  answered Jan 19 at 12:51









                  Dr. Sonnhard GraubnerDr. Sonnhard Graubner

                  76.7k42866




                  76.7k42866






























                      draft saved

                      draft discarded




















































                      Thanks for contributing an answer to Mathematics Stack Exchange!


                      • Please be sure to answer the question. Provide details and share your research!

                      But avoid



                      • Asking for help, clarification, or responding to other answers.

                      • Making statements based on opinion; back them up with references or personal experience.


                      Use MathJax to format equations. MathJax reference.


                      To learn more, see our tips on writing great answers.




                      draft saved


                      draft discarded














                      StackExchange.ready(
                      function () {
                      StackExchange.openid.initPostLogin('.new-post-login', 'https%3a%2f%2fmath.stackexchange.com%2fquestions%2f3079311%2fcomputation-of-a-double-exponential-integral%23new-answer', 'question_page');
                      }
                      );

                      Post as a guest















                      Required, but never shown





















































                      Required, but never shown














                      Required, but never shown












                      Required, but never shown







                      Required, but never shown

































                      Required, but never shown














                      Required, but never shown












                      Required, but never shown







                      Required, but never shown







                      Popular posts from this blog

                      Can a sorcerer learn a 5th-level spell early by creating spell slots using the Font of Magic feature?

                      Does disintegrating a polymorphed enemy still kill it after the 2018 errata?

                      A Topological Invariant for $pi_3(U(n))$